- PowerScore Staff
- Posts: 5972
- Joined: Mar 25, 2011
- Wed Feb 28, 2018 2:21 pm
#44058
Complete Question Explanation
(The complete setup for this game can be found here: lsat/viewtopic.php?t=16041)
The correct answer choice is (D)
Answer choice (A) is incorrect because G is selected but K is not.
Answer choice (B) is incorrect because J is selected but M is not.
Answer choice (C) is incorrect because only one tenant is selected, a violation of the first rule.
Answer choice (E) is incorrect because both M and P are selected, a violation of the last rule.
Thus, answer choice (D) is the correct answer.
(The complete setup for this game can be found here: lsat/viewtopic.php?t=16041)
The correct answer choice is (D)
Answer choice (A) is incorrect because G is selected but K is not.
Answer choice (B) is incorrect because J is selected but M is not.
Answer choice (C) is incorrect because only one tenant is selected, a violation of the first rule.
Answer choice (E) is incorrect because both M and P are selected, a violation of the last rule.
Thus, answer choice (D) is the correct answer.
Dave Killoran
PowerScore Test Preparation
Follow me on X/Twitter at http://twitter.com/DaveKilloran
My LSAT Articles: http://blog.powerscore.com/lsat/author/dave-killoran
PowerScore Podcast: http://www.powerscore.com/lsat/podcast/
PowerScore Test Preparation
Follow me on X/Twitter at http://twitter.com/DaveKilloran
My LSAT Articles: http://blog.powerscore.com/lsat/author/dave-killoran
PowerScore Podcast: http://www.powerscore.com/lsat/podcast/